How to solve this Diophantine equation?












4














Can anyone say how one can find solutions to the Diophantine equation $$x^3+y^4=z^2$$ in General? Only a few triples of numbers have been found, and most likely this equation has infinitely many solutions.



Examples of triples: $(6,5,29),(2,1,3),(9,6,45)$...










share|cite|improve this question









New contributor




Yan Dashkow is a new contributor to this site. Take care in asking for clarification, commenting, and answering.
Check out our Code of Conduct.

























    4














    Can anyone say how one can find solutions to the Diophantine equation $$x^3+y^4=z^2$$ in General? Only a few triples of numbers have been found, and most likely this equation has infinitely many solutions.



    Examples of triples: $(6,5,29),(2,1,3),(9,6,45)$...










    share|cite|improve this question









    New contributor




    Yan Dashkow is a new contributor to this site. Take care in asking for clarification, commenting, and answering.
    Check out our Code of Conduct.























      4












      4








      4


      2





      Can anyone say how one can find solutions to the Diophantine equation $$x^3+y^4=z^2$$ in General? Only a few triples of numbers have been found, and most likely this equation has infinitely many solutions.



      Examples of triples: $(6,5,29),(2,1,3),(9,6,45)$...










      share|cite|improve this question









      New contributor




      Yan Dashkow is a new contributor to this site. Take care in asking for clarification, commenting, and answering.
      Check out our Code of Conduct.











      Can anyone say how one can find solutions to the Diophantine equation $$x^3+y^4=z^2$$ in General? Only a few triples of numbers have been found, and most likely this equation has infinitely many solutions.



      Examples of triples: $(6,5,29),(2,1,3),(9,6,45)$...







      calculus diophantine-equations natural-numbers






      share|cite|improve this question









      New contributor




      Yan Dashkow is a new contributor to this site. Take care in asking for clarification, commenting, and answering.
      Check out our Code of Conduct.











      share|cite|improve this question









      New contributor




      Yan Dashkow is a new contributor to this site. Take care in asking for clarification, commenting, and answering.
      Check out our Code of Conduct.









      share|cite|improve this question




      share|cite|improve this question








      edited 2 days ago









      Martín Vacas Vignolo

      3,710623




      3,710623






      New contributor




      Yan Dashkow is a new contributor to this site. Take care in asking for clarification, commenting, and answering.
      Check out our Code of Conduct.









      asked 2 days ago









      Yan Dashkow

      241




      241




      New contributor




      Yan Dashkow is a new contributor to this site. Take care in asking for clarification, commenting, and answering.
      Check out our Code of Conduct.





      New contributor





      Yan Dashkow is a new contributor to this site. Take care in asking for clarification, commenting, and answering.
      Check out our Code of Conduct.






      Yan Dashkow is a new contributor to this site. Take care in asking for clarification, commenting, and answering.
      Check out our Code of Conduct.






















          5 Answers
          5






          active

          oldest

          votes


















          4














          This is a case of the generalized Fermat equation
          $$
          x^p+y^q=z^r.
          $$

          For $(p,q,r)=(3,4,2)$ we have $frac{1}{p}+frac{1}{q}+frac{1}{r}>1$, which is the spherical case. Here we have infinitely many integer solutions for this triple. The solutions are given by a finite set of polynomial parametrisations of the equation, see the following paper:



          F. Beukers, The diophantine equation $Ax^p + By^q = Cz^r$, Duke Math.J. 91(1998), 61-88.



          Further Reference: The generalized Fermat equation.






          share|cite|improve this answer































            3














            Here is one simple parameterization. We have,



            $$x^4 +(y^2-1)^3 = (y^3+3y)^2$$



            given the Pell equation $x^2-3y^2 =1$.






            share|cite|improve this answer























            • @AlexD $r=2$ in this question.
              – Dietrich Burde
              2 days ago










            • Thanks, @DietrichBurde. Let me try again: A solution to the problem here is a tuple of 3 numbers. But the parameterization here only relates $x$ and $y$. What about $z$?
              – Alex D
              yesterday










            • @AlexD We have $X^4+Y^3=Z^2$ and $X$,$Y$,$Z$ depend on $x$ and $y$ only. So no $z$ in here, only two parameters $x,y$ here for an equation in three variables $X,Y,Z$.
              – Dietrich Burde
              yesterday












            • Well, @DietrichBurde, you can find out more, but how can you find a general formula (solve this equation in general form), expressed in arbitrary numbers (for example) m, n, k? For example, find the top three numbers (6, 5, 29)?
              – Yan Dashkow
              yesterday










            • @YanDashkow We find them using one of the parametrizations, see Beukers article.
              – Dietrich Burde
              yesterday





















            0














            Above equation shown below has parameterization:



            $x^3+y^4=z^2$



            The below parameterization has no restriction such as the



            Pell equation condition demonstrated by Tito Piezas.



            $x=(p)^2(-q)^3$



            $y=(p)(q)^2(k-1)$



            $z=(p)^2(q)^4(2k-3)$



            where, $p=(k-2)$ and $q=(k^2-2)$



            For $k=3$ we get :
            $(-343)^3+(98)^4=(7203)^2$






            share|cite|improve this answer








            New contributor




            Sam is a new contributor to this site. Take care in asking for clarification, commenting, and answering.
            Check out our Code of Conduct.


















            • I have a question: through which k you can find three solutions (x, y, z) — (6,5,29)?
              – Yan Dashkow
              yesterday





















            0














            "OP" asked for parametric solution for $(x,y,z)=(6,5,29)$ in $x^3+y^4=z^2$



            Solution is:



            $x=3p^3(8k^2-40k+50)$



            $y=p^2(20k^2-104k+135)$



            $z=p^4(2k-5)^2(116k^2-540k+621)$



            Where, $p=(4k^2-27)$



            For $k=(13/5)$, we get after removing common factors:



            $6^3+5^4=29^2$






            share|cite|improve this answer








            New contributor




            Sam is a new contributor to this site. Take care in asking for clarification, commenting, and answering.
            Check out our Code of Conduct.


















            • Wait, why can't k be whole?
              – Yan Dashkow
              yesterday



















            0














            "OP" enquired about integer coefficent's for the parametric



            solution for the equation $(x^2+y^4=z^2)$. "OP" just needs



            to substitute $k=(m/n)$ in the parametrization & the resulting



            solution after removing common factors is given below.



            $x=6(u^3)(v^2)$



            $y=(u^2)(v)(10m-27n)$



            $z=(u^4)(v^2)(116m^2-540mn+621n^2)$



            And $u=(4m^2-27n^2)$ & $v=(2m-5n)$



            For $(m,n)=(13,5)$ we get:



            $6^3+5^4=29^2$






            share|cite|improve this answer








            New contributor




            Sam is a new contributor to this site. Take care in asking for clarification, commenting, and answering.
            Check out our Code of Conduct.


















              Your Answer





              StackExchange.ifUsing("editor", function () {
              return StackExchange.using("mathjaxEditing", function () {
              StackExchange.MarkdownEditor.creationCallbacks.add(function (editor, postfix) {
              StackExchange.mathjaxEditing.prepareWmdForMathJax(editor, postfix, [["$", "$"], ["\\(","\\)"]]);
              });
              });
              }, "mathjax-editing");

              StackExchange.ready(function() {
              var channelOptions = {
              tags: "".split(" "),
              id: "69"
              };
              initTagRenderer("".split(" "), "".split(" "), channelOptions);

              StackExchange.using("externalEditor", function() {
              // Have to fire editor after snippets, if snippets enabled
              if (StackExchange.settings.snippets.snippetsEnabled) {
              StackExchange.using("snippets", function() {
              createEditor();
              });
              }
              else {
              createEditor();
              }
              });

              function createEditor() {
              StackExchange.prepareEditor({
              heartbeatType: 'answer',
              autoActivateHeartbeat: false,
              convertImagesToLinks: true,
              noModals: true,
              showLowRepImageUploadWarning: true,
              reputationToPostImages: 10,
              bindNavPrevention: true,
              postfix: "",
              imageUploader: {
              brandingHtml: "Powered by u003ca class="icon-imgur-white" href="https://imgur.com/"u003eu003c/au003e",
              contentPolicyHtml: "User contributions licensed under u003ca href="https://creativecommons.org/licenses/by-sa/3.0/"u003ecc by-sa 3.0 with attribution requiredu003c/au003e u003ca href="https://stackoverflow.com/legal/content-policy"u003e(content policy)u003c/au003e",
              allowUrls: true
              },
              noCode: true, onDemand: true,
              discardSelector: ".discard-answer"
              ,immediatelyShowMarkdownHelp:true
              });


              }
              });






              Yan Dashkow is a new contributor. Be nice, and check out our Code of Conduct.










              draft saved

              draft discarded


















              StackExchange.ready(
              function () {
              StackExchange.openid.initPostLogin('.new-post-login', 'https%3a%2f%2fmath.stackexchange.com%2fquestions%2f3055812%2fhow-to-solve-this-diophantine-equation%23new-answer', 'question_page');
              }
              );

              Post as a guest















              Required, but never shown

























              5 Answers
              5






              active

              oldest

              votes








              5 Answers
              5






              active

              oldest

              votes









              active

              oldest

              votes






              active

              oldest

              votes









              4














              This is a case of the generalized Fermat equation
              $$
              x^p+y^q=z^r.
              $$

              For $(p,q,r)=(3,4,2)$ we have $frac{1}{p}+frac{1}{q}+frac{1}{r}>1$, which is the spherical case. Here we have infinitely many integer solutions for this triple. The solutions are given by a finite set of polynomial parametrisations of the equation, see the following paper:



              F. Beukers, The diophantine equation $Ax^p + By^q = Cz^r$, Duke Math.J. 91(1998), 61-88.



              Further Reference: The generalized Fermat equation.






              share|cite|improve this answer




























                4














                This is a case of the generalized Fermat equation
                $$
                x^p+y^q=z^r.
                $$

                For $(p,q,r)=(3,4,2)$ we have $frac{1}{p}+frac{1}{q}+frac{1}{r}>1$, which is the spherical case. Here we have infinitely many integer solutions for this triple. The solutions are given by a finite set of polynomial parametrisations of the equation, see the following paper:



                F. Beukers, The diophantine equation $Ax^p + By^q = Cz^r$, Duke Math.J. 91(1998), 61-88.



                Further Reference: The generalized Fermat equation.






                share|cite|improve this answer


























                  4












                  4








                  4






                  This is a case of the generalized Fermat equation
                  $$
                  x^p+y^q=z^r.
                  $$

                  For $(p,q,r)=(3,4,2)$ we have $frac{1}{p}+frac{1}{q}+frac{1}{r}>1$, which is the spherical case. Here we have infinitely many integer solutions for this triple. The solutions are given by a finite set of polynomial parametrisations of the equation, see the following paper:



                  F. Beukers, The diophantine equation $Ax^p + By^q = Cz^r$, Duke Math.J. 91(1998), 61-88.



                  Further Reference: The generalized Fermat equation.






                  share|cite|improve this answer














                  This is a case of the generalized Fermat equation
                  $$
                  x^p+y^q=z^r.
                  $$

                  For $(p,q,r)=(3,4,2)$ we have $frac{1}{p}+frac{1}{q}+frac{1}{r}>1$, which is the spherical case. Here we have infinitely many integer solutions for this triple. The solutions are given by a finite set of polynomial parametrisations of the equation, see the following paper:



                  F. Beukers, The diophantine equation $Ax^p + By^q = Cz^r$, Duke Math.J. 91(1998), 61-88.



                  Further Reference: The generalized Fermat equation.







                  share|cite|improve this answer














                  share|cite|improve this answer



                  share|cite|improve this answer








                  edited 2 days ago

























                  answered 2 days ago









                  Dietrich Burde

                  77.6k64386




                  77.6k64386























                      3














                      Here is one simple parameterization. We have,



                      $$x^4 +(y^2-1)^3 = (y^3+3y)^2$$



                      given the Pell equation $x^2-3y^2 =1$.






                      share|cite|improve this answer























                      • @AlexD $r=2$ in this question.
                        – Dietrich Burde
                        2 days ago










                      • Thanks, @DietrichBurde. Let me try again: A solution to the problem here is a tuple of 3 numbers. But the parameterization here only relates $x$ and $y$. What about $z$?
                        – Alex D
                        yesterday










                      • @AlexD We have $X^4+Y^3=Z^2$ and $X$,$Y$,$Z$ depend on $x$ and $y$ only. So no $z$ in here, only two parameters $x,y$ here for an equation in three variables $X,Y,Z$.
                        – Dietrich Burde
                        yesterday












                      • Well, @DietrichBurde, you can find out more, but how can you find a general formula (solve this equation in general form), expressed in arbitrary numbers (for example) m, n, k? For example, find the top three numbers (6, 5, 29)?
                        – Yan Dashkow
                        yesterday










                      • @YanDashkow We find them using one of the parametrizations, see Beukers article.
                        – Dietrich Burde
                        yesterday


















                      3














                      Here is one simple parameterization. We have,



                      $$x^4 +(y^2-1)^3 = (y^3+3y)^2$$



                      given the Pell equation $x^2-3y^2 =1$.






                      share|cite|improve this answer























                      • @AlexD $r=2$ in this question.
                        – Dietrich Burde
                        2 days ago










                      • Thanks, @DietrichBurde. Let me try again: A solution to the problem here is a tuple of 3 numbers. But the parameterization here only relates $x$ and $y$. What about $z$?
                        – Alex D
                        yesterday










                      • @AlexD We have $X^4+Y^3=Z^2$ and $X$,$Y$,$Z$ depend on $x$ and $y$ only. So no $z$ in here, only two parameters $x,y$ here for an equation in three variables $X,Y,Z$.
                        – Dietrich Burde
                        yesterday












                      • Well, @DietrichBurde, you can find out more, but how can you find a general formula (solve this equation in general form), expressed in arbitrary numbers (for example) m, n, k? For example, find the top three numbers (6, 5, 29)?
                        – Yan Dashkow
                        yesterday










                      • @YanDashkow We find them using one of the parametrizations, see Beukers article.
                        – Dietrich Burde
                        yesterday
















                      3












                      3








                      3






                      Here is one simple parameterization. We have,



                      $$x^4 +(y^2-1)^3 = (y^3+3y)^2$$



                      given the Pell equation $x^2-3y^2 =1$.






                      share|cite|improve this answer














                      Here is one simple parameterization. We have,



                      $$x^4 +(y^2-1)^3 = (y^3+3y)^2$$



                      given the Pell equation $x^2-3y^2 =1$.







                      share|cite|improve this answer














                      share|cite|improve this answer



                      share|cite|improve this answer








                      edited 2 days ago

























                      answered 2 days ago









                      Tito Piezas III

                      26.8k365169




                      26.8k365169












                      • @AlexD $r=2$ in this question.
                        – Dietrich Burde
                        2 days ago










                      • Thanks, @DietrichBurde. Let me try again: A solution to the problem here is a tuple of 3 numbers. But the parameterization here only relates $x$ and $y$. What about $z$?
                        – Alex D
                        yesterday










                      • @AlexD We have $X^4+Y^3=Z^2$ and $X$,$Y$,$Z$ depend on $x$ and $y$ only. So no $z$ in here, only two parameters $x,y$ here for an equation in three variables $X,Y,Z$.
                        – Dietrich Burde
                        yesterday












                      • Well, @DietrichBurde, you can find out more, but how can you find a general formula (solve this equation in general form), expressed in arbitrary numbers (for example) m, n, k? For example, find the top three numbers (6, 5, 29)?
                        – Yan Dashkow
                        yesterday










                      • @YanDashkow We find them using one of the parametrizations, see Beukers article.
                        – Dietrich Burde
                        yesterday




















                      • @AlexD $r=2$ in this question.
                        – Dietrich Burde
                        2 days ago










                      • Thanks, @DietrichBurde. Let me try again: A solution to the problem here is a tuple of 3 numbers. But the parameterization here only relates $x$ and $y$. What about $z$?
                        – Alex D
                        yesterday










                      • @AlexD We have $X^4+Y^3=Z^2$ and $X$,$Y$,$Z$ depend on $x$ and $y$ only. So no $z$ in here, only two parameters $x,y$ here for an equation in three variables $X,Y,Z$.
                        – Dietrich Burde
                        yesterday












                      • Well, @DietrichBurde, you can find out more, but how can you find a general formula (solve this equation in general form), expressed in arbitrary numbers (for example) m, n, k? For example, find the top three numbers (6, 5, 29)?
                        – Yan Dashkow
                        yesterday










                      • @YanDashkow We find them using one of the parametrizations, see Beukers article.
                        – Dietrich Burde
                        yesterday


















                      @AlexD $r=2$ in this question.
                      – Dietrich Burde
                      2 days ago




                      @AlexD $r=2$ in this question.
                      – Dietrich Burde
                      2 days ago












                      Thanks, @DietrichBurde. Let me try again: A solution to the problem here is a tuple of 3 numbers. But the parameterization here only relates $x$ and $y$. What about $z$?
                      – Alex D
                      yesterday




                      Thanks, @DietrichBurde. Let me try again: A solution to the problem here is a tuple of 3 numbers. But the parameterization here only relates $x$ and $y$. What about $z$?
                      – Alex D
                      yesterday












                      @AlexD We have $X^4+Y^3=Z^2$ and $X$,$Y$,$Z$ depend on $x$ and $y$ only. So no $z$ in here, only two parameters $x,y$ here for an equation in three variables $X,Y,Z$.
                      – Dietrich Burde
                      yesterday






                      @AlexD We have $X^4+Y^3=Z^2$ and $X$,$Y$,$Z$ depend on $x$ and $y$ only. So no $z$ in here, only two parameters $x,y$ here for an equation in three variables $X,Y,Z$.
                      – Dietrich Burde
                      yesterday














                      Well, @DietrichBurde, you can find out more, but how can you find a general formula (solve this equation in general form), expressed in arbitrary numbers (for example) m, n, k? For example, find the top three numbers (6, 5, 29)?
                      – Yan Dashkow
                      yesterday




                      Well, @DietrichBurde, you can find out more, but how can you find a general formula (solve this equation in general form), expressed in arbitrary numbers (for example) m, n, k? For example, find the top three numbers (6, 5, 29)?
                      – Yan Dashkow
                      yesterday












                      @YanDashkow We find them using one of the parametrizations, see Beukers article.
                      – Dietrich Burde
                      yesterday






                      @YanDashkow We find them using one of the parametrizations, see Beukers article.
                      – Dietrich Burde
                      yesterday













                      0














                      Above equation shown below has parameterization:



                      $x^3+y^4=z^2$



                      The below parameterization has no restriction such as the



                      Pell equation condition demonstrated by Tito Piezas.



                      $x=(p)^2(-q)^3$



                      $y=(p)(q)^2(k-1)$



                      $z=(p)^2(q)^4(2k-3)$



                      where, $p=(k-2)$ and $q=(k^2-2)$



                      For $k=3$ we get :
                      $(-343)^3+(98)^4=(7203)^2$






                      share|cite|improve this answer








                      New contributor




                      Sam is a new contributor to this site. Take care in asking for clarification, commenting, and answering.
                      Check out our Code of Conduct.


















                      • I have a question: through which k you can find three solutions (x, y, z) — (6,5,29)?
                        – Yan Dashkow
                        yesterday


















                      0














                      Above equation shown below has parameterization:



                      $x^3+y^4=z^2$



                      The below parameterization has no restriction such as the



                      Pell equation condition demonstrated by Tito Piezas.



                      $x=(p)^2(-q)^3$



                      $y=(p)(q)^2(k-1)$



                      $z=(p)^2(q)^4(2k-3)$



                      where, $p=(k-2)$ and $q=(k^2-2)$



                      For $k=3$ we get :
                      $(-343)^3+(98)^4=(7203)^2$






                      share|cite|improve this answer








                      New contributor




                      Sam is a new contributor to this site. Take care in asking for clarification, commenting, and answering.
                      Check out our Code of Conduct.


















                      • I have a question: through which k you can find three solutions (x, y, z) — (6,5,29)?
                        – Yan Dashkow
                        yesterday
















                      0












                      0








                      0






                      Above equation shown below has parameterization:



                      $x^3+y^4=z^2$



                      The below parameterization has no restriction such as the



                      Pell equation condition demonstrated by Tito Piezas.



                      $x=(p)^2(-q)^3$



                      $y=(p)(q)^2(k-1)$



                      $z=(p)^2(q)^4(2k-3)$



                      where, $p=(k-2)$ and $q=(k^2-2)$



                      For $k=3$ we get :
                      $(-343)^3+(98)^4=(7203)^2$






                      share|cite|improve this answer








                      New contributor




                      Sam is a new contributor to this site. Take care in asking for clarification, commenting, and answering.
                      Check out our Code of Conduct.









                      Above equation shown below has parameterization:



                      $x^3+y^4=z^2$



                      The below parameterization has no restriction such as the



                      Pell equation condition demonstrated by Tito Piezas.



                      $x=(p)^2(-q)^3$



                      $y=(p)(q)^2(k-1)$



                      $z=(p)^2(q)^4(2k-3)$



                      where, $p=(k-2)$ and $q=(k^2-2)$



                      For $k=3$ we get :
                      $(-343)^3+(98)^4=(7203)^2$







                      share|cite|improve this answer








                      New contributor




                      Sam is a new contributor to this site. Take care in asking for clarification, commenting, and answering.
                      Check out our Code of Conduct.









                      share|cite|improve this answer



                      share|cite|improve this answer






                      New contributor




                      Sam is a new contributor to this site. Take care in asking for clarification, commenting, and answering.
                      Check out our Code of Conduct.









                      answered 2 days ago









                      Sam

                      1




                      1




                      New contributor




                      Sam is a new contributor to this site. Take care in asking for clarification, commenting, and answering.
                      Check out our Code of Conduct.





                      New contributor





                      Sam is a new contributor to this site. Take care in asking for clarification, commenting, and answering.
                      Check out our Code of Conduct.






                      Sam is a new contributor to this site. Take care in asking for clarification, commenting, and answering.
                      Check out our Code of Conduct.












                      • I have a question: through which k you can find three solutions (x, y, z) — (6,5,29)?
                        – Yan Dashkow
                        yesterday




















                      • I have a question: through which k you can find three solutions (x, y, z) — (6,5,29)?
                        – Yan Dashkow
                        yesterday


















                      I have a question: through which k you can find three solutions (x, y, z) — (6,5,29)?
                      – Yan Dashkow
                      yesterday






                      I have a question: through which k you can find three solutions (x, y, z) — (6,5,29)?
                      – Yan Dashkow
                      yesterday













                      0














                      "OP" asked for parametric solution for $(x,y,z)=(6,5,29)$ in $x^3+y^4=z^2$



                      Solution is:



                      $x=3p^3(8k^2-40k+50)$



                      $y=p^2(20k^2-104k+135)$



                      $z=p^4(2k-5)^2(116k^2-540k+621)$



                      Where, $p=(4k^2-27)$



                      For $k=(13/5)$, we get after removing common factors:



                      $6^3+5^4=29^2$






                      share|cite|improve this answer








                      New contributor




                      Sam is a new contributor to this site. Take care in asking for clarification, commenting, and answering.
                      Check out our Code of Conduct.


















                      • Wait, why can't k be whole?
                        – Yan Dashkow
                        yesterday
















                      0














                      "OP" asked for parametric solution for $(x,y,z)=(6,5,29)$ in $x^3+y^4=z^2$



                      Solution is:



                      $x=3p^3(8k^2-40k+50)$



                      $y=p^2(20k^2-104k+135)$



                      $z=p^4(2k-5)^2(116k^2-540k+621)$



                      Where, $p=(4k^2-27)$



                      For $k=(13/5)$, we get after removing common factors:



                      $6^3+5^4=29^2$






                      share|cite|improve this answer








                      New contributor




                      Sam is a new contributor to this site. Take care in asking for clarification, commenting, and answering.
                      Check out our Code of Conduct.


















                      • Wait, why can't k be whole?
                        – Yan Dashkow
                        yesterday














                      0












                      0








                      0






                      "OP" asked for parametric solution for $(x,y,z)=(6,5,29)$ in $x^3+y^4=z^2$



                      Solution is:



                      $x=3p^3(8k^2-40k+50)$



                      $y=p^2(20k^2-104k+135)$



                      $z=p^4(2k-5)^2(116k^2-540k+621)$



                      Where, $p=(4k^2-27)$



                      For $k=(13/5)$, we get after removing common factors:



                      $6^3+5^4=29^2$






                      share|cite|improve this answer








                      New contributor




                      Sam is a new contributor to this site. Take care in asking for clarification, commenting, and answering.
                      Check out our Code of Conduct.









                      "OP" asked for parametric solution for $(x,y,z)=(6,5,29)$ in $x^3+y^4=z^2$



                      Solution is:



                      $x=3p^3(8k^2-40k+50)$



                      $y=p^2(20k^2-104k+135)$



                      $z=p^4(2k-5)^2(116k^2-540k+621)$



                      Where, $p=(4k^2-27)$



                      For $k=(13/5)$, we get after removing common factors:



                      $6^3+5^4=29^2$







                      share|cite|improve this answer








                      New contributor




                      Sam is a new contributor to this site. Take care in asking for clarification, commenting, and answering.
                      Check out our Code of Conduct.









                      share|cite|improve this answer



                      share|cite|improve this answer






                      New contributor




                      Sam is a new contributor to this site. Take care in asking for clarification, commenting, and answering.
                      Check out our Code of Conduct.









                      answered yesterday









                      Sam

                      1




                      1




                      New contributor




                      Sam is a new contributor to this site. Take care in asking for clarification, commenting, and answering.
                      Check out our Code of Conduct.





                      New contributor





                      Sam is a new contributor to this site. Take care in asking for clarification, commenting, and answering.
                      Check out our Code of Conduct.






                      Sam is a new contributor to this site. Take care in asking for clarification, commenting, and answering.
                      Check out our Code of Conduct.












                      • Wait, why can't k be whole?
                        – Yan Dashkow
                        yesterday


















                      • Wait, why can't k be whole?
                        – Yan Dashkow
                        yesterday
















                      Wait, why can't k be whole?
                      – Yan Dashkow
                      yesterday




                      Wait, why can't k be whole?
                      – Yan Dashkow
                      yesterday











                      0














                      "OP" enquired about integer coefficent's for the parametric



                      solution for the equation $(x^2+y^4=z^2)$. "OP" just needs



                      to substitute $k=(m/n)$ in the parametrization & the resulting



                      solution after removing common factors is given below.



                      $x=6(u^3)(v^2)$



                      $y=(u^2)(v)(10m-27n)$



                      $z=(u^4)(v^2)(116m^2-540mn+621n^2)$



                      And $u=(4m^2-27n^2)$ & $v=(2m-5n)$



                      For $(m,n)=(13,5)$ we get:



                      $6^3+5^4=29^2$






                      share|cite|improve this answer








                      New contributor




                      Sam is a new contributor to this site. Take care in asking for clarification, commenting, and answering.
                      Check out our Code of Conduct.























                        0














                        "OP" enquired about integer coefficent's for the parametric



                        solution for the equation $(x^2+y^4=z^2)$. "OP" just needs



                        to substitute $k=(m/n)$ in the parametrization & the resulting



                        solution after removing common factors is given below.



                        $x=6(u^3)(v^2)$



                        $y=(u^2)(v)(10m-27n)$



                        $z=(u^4)(v^2)(116m^2-540mn+621n^2)$



                        And $u=(4m^2-27n^2)$ & $v=(2m-5n)$



                        For $(m,n)=(13,5)$ we get:



                        $6^3+5^4=29^2$






                        share|cite|improve this answer








                        New contributor




                        Sam is a new contributor to this site. Take care in asking for clarification, commenting, and answering.
                        Check out our Code of Conduct.





















                          0












                          0








                          0






                          "OP" enquired about integer coefficent's for the parametric



                          solution for the equation $(x^2+y^4=z^2)$. "OP" just needs



                          to substitute $k=(m/n)$ in the parametrization & the resulting



                          solution after removing common factors is given below.



                          $x=6(u^3)(v^2)$



                          $y=(u^2)(v)(10m-27n)$



                          $z=(u^4)(v^2)(116m^2-540mn+621n^2)$



                          And $u=(4m^2-27n^2)$ & $v=(2m-5n)$



                          For $(m,n)=(13,5)$ we get:



                          $6^3+5^4=29^2$






                          share|cite|improve this answer








                          New contributor




                          Sam is a new contributor to this site. Take care in asking for clarification, commenting, and answering.
                          Check out our Code of Conduct.









                          "OP" enquired about integer coefficent's for the parametric



                          solution for the equation $(x^2+y^4=z^2)$. "OP" just needs



                          to substitute $k=(m/n)$ in the parametrization & the resulting



                          solution after removing common factors is given below.



                          $x=6(u^3)(v^2)$



                          $y=(u^2)(v)(10m-27n)$



                          $z=(u^4)(v^2)(116m^2-540mn+621n^2)$



                          And $u=(4m^2-27n^2)$ & $v=(2m-5n)$



                          For $(m,n)=(13,5)$ we get:



                          $6^3+5^4=29^2$







                          share|cite|improve this answer








                          New contributor




                          Sam is a new contributor to this site. Take care in asking for clarification, commenting, and answering.
                          Check out our Code of Conduct.









                          share|cite|improve this answer



                          share|cite|improve this answer






                          New contributor




                          Sam is a new contributor to this site. Take care in asking for clarification, commenting, and answering.
                          Check out our Code of Conduct.









                          answered 21 hours ago









                          Sam

                          1




                          1




                          New contributor




                          Sam is a new contributor to this site. Take care in asking for clarification, commenting, and answering.
                          Check out our Code of Conduct.





                          New contributor





                          Sam is a new contributor to this site. Take care in asking for clarification, commenting, and answering.
                          Check out our Code of Conduct.






                          Sam is a new contributor to this site. Take care in asking for clarification, commenting, and answering.
                          Check out our Code of Conduct.






















                              Yan Dashkow is a new contributor. Be nice, and check out our Code of Conduct.










                              draft saved

                              draft discarded


















                              Yan Dashkow is a new contributor. Be nice, and check out our Code of Conduct.













                              Yan Dashkow is a new contributor. Be nice, and check out our Code of Conduct.












                              Yan Dashkow is a new contributor. Be nice, and check out our Code of Conduct.
















                              Thanks for contributing an answer to Mathematics Stack Exchange!


                              • Please be sure to answer the question. Provide details and share your research!

                              But avoid



                              • Asking for help, clarification, or responding to other answers.

                              • Making statements based on opinion; back them up with references or personal experience.


                              Use MathJax to format equations. MathJax reference.


                              To learn more, see our tips on writing great answers.





                              Some of your past answers have not been well-received, and you're in danger of being blocked from answering.


                              Please pay close attention to the following guidance:


                              • Please be sure to answer the question. Provide details and share your research!

                              But avoid



                              • Asking for help, clarification, or responding to other answers.

                              • Making statements based on opinion; back them up with references or personal experience.


                              To learn more, see our tips on writing great answers.




                              draft saved


                              draft discarded














                              StackExchange.ready(
                              function () {
                              StackExchange.openid.initPostLogin('.new-post-login', 'https%3a%2f%2fmath.stackexchange.com%2fquestions%2f3055812%2fhow-to-solve-this-diophantine-equation%23new-answer', 'question_page');
                              }
                              );

                              Post as a guest















                              Required, but never shown





















































                              Required, but never shown














                              Required, but never shown












                              Required, but never shown







                              Required, but never shown

































                              Required, but never shown














                              Required, but never shown












                              Required, but never shown







                              Required, but never shown







                              Popular posts from this blog

                              If I really need a card on my start hand, how many mulligans make sense? [duplicate]

                              Alcedinidae

                              Can an atomic nucleus contain both particles and antiparticles? [duplicate]